LSAT and Law School Admissions Forum

Get expert LSAT preparation and law school admissions advice from PowerScore Test Preparation.

 Administrator
PowerScore Staff
  • PowerScore Staff
  • Posts: 8917
  • Joined: Feb 02, 2011
|
#36409
Complete Question Explanation

Assumption. The correct answer choice is (B)

The stimulus points out that art theft is on the rise, and that most stolen art is sold to wealthy
collectors. The author then argues that since thieves steal what their customers are most interested in
buying, security should focus on protecting museums’ most valuable pieces.

If you are having trouble identifying the conclusion, note the use of the conclusion/premise indicator
form “Consequently, since…” The conclusion is “museums ought to focus more of their security on
their most valuable pieces.”

The conclusion of the argument introduces a new idea: “most valuable pieces.” Where does the
basis for this new idea come from? There is no concrete statement about the value of the pieces,
so obviously the author makes a leap to arrive at this concept, and in an Assumption question it is
highly likely that the correct answer will address this new idea (notably, only answer choices (A),
(B), and (D) address the value of art, and this means that answer choices (C) and (E) are unlikely to
be correct). The premise just prior to the conclusion also includes a new idea: “what their customers
are most interested in buying.” This new idea will also likely be addressed in the correct answer, and
notably only answer choice (B) addresses this idea. Thus, just based on an analysis of the pieces of
this argument, answer choice (B) jumps out as the most likely correct answer because it connect the
“new ideas” in the premise and conclusion. Remember, to excel on the LSAT you must understand
how the test makers operate and take advantage of the patterns they use.

Answer choice (A): This response would serve to undermine the argument, so this choice is wrong.

Answer choice (B): This is the correct answer choice. This Supporter answer establishes a
connection between value of the art and what the collectors want, and that fi lls the gap in the author’s
argument.

Note the proper negation of this answer: “Art pieces that are not very valuable are very much in
demand by wealthy private collectors.” As this negation would undermine the author’s argument, we
know from the Assumption Negation Technique that this is the correct answer.

Answer choice (C): This response is irrelevant because the stimulus does not concern adding
security, but refocusing it. In any case, the general need for security is neither essential nor helpful in
establishing that a refocusing of security is a good plan.

Answer choice (D): This was the most popular incorrect answer. The author’s argument does not
depend on the idea that most museums equally protect all their art, just that the most valuable art in
some museums is not maximally protected.

Consider the negation of this answer: “Most museums provide different amounts of security for
valuable and not-so-valuable art.” Does this statement undermine the argument? No, because even if
the security currently focuses more on valuable items than on non-valuable items, the author could
claim it is a good idea to focus even more on those items. Since the negation of this answer choice
does not undermine the author’s argument, we know from the Assumption Negation Technique that
this is an incorrect answer.

Answer choice (E): What wealthy private collectors do with their art after they buy it is beyond the
scope of this argument, and so this choice is wrong.
 voodoochild
  • Posts: 185
  • Joined: Apr 25, 2012
|
#8780
Experts,
I was down to B and D, and ended up choosing D on this one. Can you please help me in ruling out D?

Premise - Most stolen art is sold to wealthy collectors
Int conclusion - Thieves steal what their customers are most interested in buying

Gap1 - connection between "stolen art" and "most interested in buying" ..i.e. thieves know what their customers are "most interested in"

Final conclusion - museums ought to focus more of their security on their most valuable pieces.

Gap2 - "most interested in buying" articles -> require more security than non valuable ones
Gap 3 - "most interested in buying" -> valuable articles.

I could find three major gaps. B matches contrapositive of Gap3. But I am not sure why D is incorrect. Any help?

Thanks
 Steve Stein
PowerScore Staff
  • PowerScore Staff
  • Posts: 1153
  • Joined: Apr 11, 2011
|
#8783
Hey voodoochild,

That's a good question. The basic argument: Most stolen art is sold to wealthy people, and thieves aim to please their customers. Thus, museums should focus more on protecting the most valuable pieces.

Since this is an Assumption question, we can use PowerScore's Assumption Negation Technique to test the answer choices--when we negate the correct answer choice, that will weaken the author's argument.

The negated version of choice B: Art pieces that are not very valuable are in high demand. That would weaken the author's conclusion about more focus on only the most valuable pieces.

The negated version of choice D: Most museums don't provide the same level of security for all of their pieces.

Keep in mind that the author's conclusion was about what museums should do, not what they actually are doing, so the negated version of choice D above would not weaken this conclusion.

I hope that's helpful! Please let me know whether that clears this one up--thanks!

~Steve
 voodoochild
  • Posts: 185
  • Joined: Apr 25, 2012
|
#8807
Steve Stein wrote:
Keep in mind that the author's conclusion was about what museums should do, not what they actually are doing, so the negated version of choice D above would not weaken this conclusion.
Steve,
I have a question. I see how negation kills D. But, if, say, a doctor recommends to exercise everyday, then don't you think the implied assumption is that the patient is not exercising?

Similarly, if the author recommends that the museums focus on increasing security for valuable possessions, then isn't it assumed that valued possessions don't have an expected level of security?

Can you please help me?

Thanks
Voodoo
 Steve Stein
PowerScore Staff
  • PowerScore Staff
  • Posts: 1153
  • Joined: Apr 11, 2011
|
#8808
Hi voodoochild,

Thanks for your response--you bring up an interesting point. Not every piece of advice, however, is necessarily a prescription for change--advice can also confirm that no change is necessary. Consider the following two scenarios:

Patient: I exercise every day.
Doctor: Well, you should exercise every day.
Patient: Nice! :-D

vs.

Patient: I don't exercise much.
Doctor: Well, you should exercise every day.
Patient: Oh, alright, I'll change my habits :hmm:

The Doctor's advice is consistent, but there's no way to determine, based on that advice, the patient's pre-existing behavior.

Interesting point! I hope that's helpful--please let me know whether that clears this one up--thanks!

~Steve
 voodoochild
  • Posts: 185
  • Joined: Apr 25, 2012
|
#8814
Steve,

Thanks for your response.

I am trying to play devil's advocate so that I can develop a critical eye that would tell me what's relevant and what's irrelevant. If this question were, say, a weakener, do you think that the fact "currently, museums provide highest level of security to costly possessions" would not impact the argument? In other words, "wealthy investors who wish to buy valuable items are kept in extremely secure place." I believe this would have no impact on the argument. Can you please confirm?

I am not sure how I can weaken the argument other than by breaking the link between "most interested in buying" articles and "highly valuable" articles. For instance, "recently, wealthy investors are turning to buy antique items that don't cost much"

Can you please confirm?

Thanks
 Steve Stein
PowerScore Staff
  • PowerScore Staff
  • Posts: 1153
  • Joined: Apr 11, 2011
|
#8815
Thanks for your response.

Again, here the author's conclusion takes the form of advice: not what is happening necessarily, but rather what the author concludes should be the case.

"If this question were, say, a weakener, do you think that the fact "currently, museums provide highest level of security to costly possessions" would not impact the argument?"

No. The fact that costly possessions already get the most security would not have any effect on the strength of the author's argument. As reflected in the doctor/patient scenario from my previous post, the patient's current behavior (just like the current state of costly item security) does nothing to weaken the doctor's argument; The doctor asserts that daily exercise is advisable--this is not affected by whether or not the patient is already exercising daily.

The way to weaken the argument would be to focus not on what museums are actually doing with regard to security, but on why focusing mostly on the most expensive items is not necessarily advisable. This would hurt the author's argument.

I hope that's helpful--please let me know--thanks!

~Steve
 voodoochild
  • Posts: 185
  • Joined: Apr 25, 2012
|
#8816
Thanks Steve!
 Kp13
  • Posts: 32
  • Joined: Jun 17, 2013
|
#10236
Hi,

I am having a hard time distinguishing answer choice B) from D). Both seem to be equally likely assumptions that the author makes. How do I tell them apart?

Thanks!
 Ron Gore
PowerScore Staff
  • PowerScore Staff
  • Posts: 220
  • Joined: May 15, 2013
|
#10243
Thank you for your question, KP13.

Because there is no apparent "rogue" or "new" information in the stimulus calling out to be linked to another part of the argument by the answer choice, this is an Assumption question of the Defender model. The task of the correct answer here is to help the stimulus eliminating an idea that could weaken the conclusion. In practical terms, what you will see in the correct answer choice is a potential weakness being raised so that it can be dismissed.

Before you can apply this approach, you need to prephrase what may be a weakness in the argument. Here, the conclusion is that "museums out to focus more of their security on their most valuable pieces." The author supports this conclusion by stating "most art is sold to wealthy private collectors," and that "thieves steal what their customers are most interested in buying."

The weakness in this argument is that there is no evidence given to support the idea that just because the customers are "wealthy private collectors," that they are most interested in buying the "most valuable pieces." An equally likely possibility, given the evidence, is that the wealthy private collectors would be interested in artwork of only low or middle-range value. We cannot assume that because the private collectors are wealthy that they are most interested in the most valuable pieces.

Choice (B), which is the correct answer, raises the prephrased weakness, that the private collectors might be more interested in those pieces that are not very valuable, but does so only to dismiss the notion: "Art pieces that are not very valuable are NOT very much in demand by wealthy private collectors." As written, this answer choice defends the conclusion against an attack on the weakness we identified. If negated, this choice would attack the conclusion, and so we can conclude it is the correct answer.

Regarding choice (D), remember that the assumption answer choice, by definition, provides information necessary for even the possibility that the conclusion is valid. For (D) to be correct, it would be necessary that "MOST museums provide the SAME amount of security for valuable and no-so-valuable art." The conclusion in the stimulus dealt with an opinion regarding what museums should do, and this information regarding what "most" museums currently do is not necessary for that conclusion to be valid.

Also, it is not necessary for the conclusion to be valid that most museums in fact offer the "SAME" amount of security for the valuable and the not-so-valuable art. Even if you were looking at this as a statement of what the current situation is and how it needs to change, which is not a permissible view of the stimulus, the current levels of security would not need to be identical for the conclusion to be valid.

Hope that helps.

Ron

Get the most out of your LSAT Prep Plus subscription.

Analyze and track your performance with our Testing and Analytics Package.